Repeated Roots and Being Relatively Prime w/Derivative

  • Thread starter Thread starter Bashyboy
  • Start date Start date
  • Tags Tags
    Prime Roots
Click For Summary
The discussion focuses on proving that a polynomial function f(x) has no repeated roots if and only if the greatest common divisor (gcd) of f(x) and its derivative f'(x) is 1. It establishes that if f(x) has repeated roots, then there exists a factor (x-a_k)^2 in f(x), which implies that x-a_k also divides f'(x), contradicting the condition that gcd(f, f') = 1. Conversely, if gcd(f, f') = 1, then f(x) cannot have repeated roots, as this would lead to a common factor. The conversation emphasizes the relationship between the structure of f(x) and its derivative in determining the nature of its roots. The reasoning is validated through the application of fundamental theorems related to polynomial factorization and differentiation.
Bashyboy
Messages
1,419
Reaction score
5

Homework Statement


Let ##f(x) = (x-a_1)...(x-a_n) \in k[x]##, where ##k## is a field. Show that ##f(x)## has no repeated roots (i.e., all the ai are distinct elements in ##k##) if and only if ##gcd(f,f')=1##, where ##f'(x)## is the derivative of ##f##

Homework Equations



##(x-a)^2 |f(x)## implies ##(x-a)|f'(x)##

##(x-a)|f(x)## and ##(x-a)|f'(x)## implies ##(x-a)^2|f(x)##

The Attempt at a Solution



First note that ##f(x) = (x-a_1)...(x-a_n)## has a repeated roots if and only if ##(x-a_k)^p## is a factor of ##f(x)## for some ##k \in \{1,...,n\}## and ##p \ge 2##.

Suppose that ##f(x)## has no repeated. Note that ##f(x) = (x-a_1)...(x-a_n)## is the prime factorization of ##f(x)##. Now if were the case that ##gcd(f,f') \neq 1##, then both ##f## and ##f'## would have a common prime factor. Since we know what ##f(x)##'s prime factors look like, we know there is a ##k## such that ##x-a_k## divides ##f'##. But the second theorem cited above implies that ##(x-a_k)^2## divides ##f(x)## and therefore it has a repeated root. Hence, ##gcd(f,f')## must be ##1##.

Now suppose that ##(f,f')=1##. If ##f(x)## had a repeated root, then ##(x-a_k)^2## would divide it, for some ##k##. But the first theorem cited above would imply ##(x-a)|f'(x)##, contradicting the fact that ##(f,f')=1##.

How does this sound?
 
Last edited:
Physics news on Phys.org
Sounds correct, although the whole work is done in the two statements you use. You could as well simply differentiate ##f(x)=\prod(x-a_k)^{n_k}## and directly see the result.
 
Question: A clock's minute hand has length 4 and its hour hand has length 3. What is the distance between the tips at the moment when it is increasing most rapidly?(Putnam Exam Question) Answer: Making assumption that both the hands moves at constant angular velocities, the answer is ## \sqrt{7} .## But don't you think this assumption is somewhat doubtful and wrong?

Similar threads

  • · Replies 12 ·
Replies
12
Views
2K
  • · Replies 1 ·
Replies
1
Views
1K
  • · Replies 1 ·
Replies
1
Views
2K
Replies
8
Views
1K
Replies
3
Views
2K
Replies
5
Views
2K
  • · Replies 3 ·
Replies
3
Views
2K
  • · Replies 3 ·
Replies
3
Views
2K
  • · Replies 18 ·
Replies
18
Views
3K
Replies
1
Views
1K